You are on page 1of 6

MATH 4603

Homework 9

Summer 2014
Due July 23th, 2014

Name:

Short Answers: (Grading: I will pick one exercise and grade it out of 10 points)
1. True or False. If false, give a counterexample. If true justify why.
(a) If f is monotone on [a, b] then f is integrable on [a, b]. (True)
Suppose f is increasing. Since f is defined at the end points, it is bounded and
there is k such that f (b) f (a) < k. Given > 0, pick a partition of [a, b] such
that (P ) < = /k. Then because f is increasing
n1
X
U (P, f ) L(P, f ) =
[f (xi+1 ) f (xi )](xi+1 xi ) < (f (b) f (a)) <
i=0

(b) If f is integrable on [a, b], then f is continuous on [a, b]. (False)


The function f : [1, 1] R, f (x) = 1 for x 0 and f (x) = 1 for x < 0 is not
continuous, but it is integrable.
(c) Let f is bounded on [a, b]. If P and Q are partitions of [a, b], then
L(P, f ) U (P, f ). (True)
Given a partition P = {x0 , x1 , , xn } of [a, b], the lower sums, L(P, f )
approximate the integral using the inf of the function on each interval [xi , xi+1 ]
and the upper sums U (P, f ) approximate the integral using the inf of the
function on each interval [xi , xi+1 ]. Therefore, L(P, f ) U (P, f ).
(d) If f is integrable on [a, b], then given any > 0 there exists a partition P of [a, b]
such that L(P, f ) > U (P, f ) . (True)
Since f is integrable, for every > 0 there is a partition P of [a, b] such that
U (P, f ) L(P, f ) < . Therefore,
U (P, f ) < L(P, f )
2. Use the Fundamental Theorem of Integral Calculus to compute the following:
Let us state the Fundamental
Theorem of Integral Calculus first: If F = f 0 and f 0 is
Rb
integrable on [a, b] then a F dx = f (b) f (a).
R3
(a) 0 (x2 x)dx.
2
3
Here F = x2 x and f = x3 x2 therefore
Z 3
9
(x2 x)dx = f (3) f (0) = (9 9/2) 0 = .
2
0
R4
(b) 2 (1 x3 x2 )dx
4
3
Here F = (1 x3 x2 ) and f = x x4 x3 therefore
Z 4
(1 x3 x2 )dx = f (4) f (2) = 78
2

MATH 4603
(c)

Homework 9

Summer 2014

R /2

x sin(x2 )dx
2)
Here F = x sin(x2 ) and f = cos(x
therefore
2
0

/2

x sin(x2 )dx = f (/2) f (0) =

cos( 2 /4) 1
+
2
2

3. Let f be a bounded function defined on the interval [a, b]. Mark each statement as
true or false, and justify your answer.
(a) The upper and lower sums of f always form a bounded set. (True)
Since f is bounded, there exists M R such that |f | < M . Therefore, given any
partition P of [a, b] we have that
M (b a) L(P, f ) U (P, f ) M (b a)
(b) If P and Q are partitions of [a, b], then P Q is a refinement of both P and Q.
(True)
Since P P Q and Q P Q, then P Q is a refinement of both P and Q.
Rb
(c) When a f dx exists, it is a unique number that lies between L(P, f ) and U (P, f )
for all partitions P of [a, b].(True)
If f is integrable, by definition we have that
Z

f dx =

f dx =
a

where

f dx
a

f dx = sup{L(P, f ) : P is a partition of[a, b]},


a

f dx = inf{L(P, f ) : P is a partition of[a, b]}.


a

If there were two numbers, A and B such that


L(P, f ) A < B U (P, f )
for any given partition P , then

Rb
a

f dx 6=

Rb
a

f dx.

(d) f is Riemann integrable iff its upper and lower sums are equal.(False)
f is Riemann integrable iff its upper and lower integrals are equal, i.e.
Z

f dx =
a

f dx =
a

f dx
a

MATH 4603

Homework 9

Summer 2014

Proof: Provide a complete proof for the following exercises (Grading: I will pick two
exercises and grade them out of 10 points).
1. Use Theorem 5.2 to prove that the function f (x) = x3 is integrable on [0, 1].
Proof. Let > 0, define = /3, and pick a partition P = {x0 , x1 , , xn } such
that
max{(xi+1 xi ) : 0 i n 1} <
Then
n1
X
U (P, f ) L(P, f ) =
(x3i+1 x3i )(xi+1 xi )

i=0
n1
X

(xi+1 xi )(x2i+1 + xi+1 xi + x2i )(xi+1 xi )

i=0
n1
X

(xi+1 xi )

< 3

i=0

< 3(1 0) = .
Therefore, f is integrable.
2. A set A [0, 1] is dense in [0, 1] iff every open interval that intersects [0, 1] contains a
point of A. Suppose f : [0,R1] R is integrable and f (x) = 0 for all x A with A
1
dense in [0, 1]. Show that 0 f dx = 0.
Proof. Suppose that
Z

Rb
a
b

f dx = sup{L(P, f ) : P is a partition of[a, b]} = A.

f dx =
a

f dx = A > 0. Since f is integrable,

In particular there exists a partition P such that


0 < A/2 < L(P, f ) A
Pn1
Moreover, we have that 0 < L(P, f ) S(P, f ), where S(P, f ) = i=0
f (ti )(xi+1 xi )
is the Riemann sum of f associated with the partition P , and ti is any number on the
interval [xi , xi+1 ]. In particular, since A is dense we can pick ti A [xi , xi+1 ]. In
this case, f (ti ) = 0 for every 0 i n 1, and we reach a contradiction:
0 < L(P, f ) S(P, f ) = 0.
Therefore,

Rb
a

f dx = A = 0.

MATH 4603

Homework 9

Summer 2014

3. Suppose g : [a, b] R is continuous except at x0 (a, b). Assume also that f is


bounded. Prove that g is Riemann integrable on [a, b].
Proof. Since f is bounded, then we do not have a vertical asymptote at x0 . We can
therefore consider g1 : [a, x0 ] R, where g1 (x) = g(x) for x 6= x0 and
g1 (x0 ) = limxx0 g(x). Now, this function is continuous on [a, x0 ] and it is therefore
Riemann integrable. A similar reasoning shows that g2 : [x0 , b] R, where
g2 (x) = g(x) for x 6= x0 and g2 (x0 ) = limxx0 g(x), is Riemann integrable.
To show that f is integrable, let P1 , P2 be any partitions of [a, x0 ] and [x0 , b]. Let
> 0 and consider P = (P1 P2 {x0 /2, x0 + /2}) \ {x0 } a partition of [a, b].
Define Q1 = P [a, x0 ] and Q2 = P [x0 + , b] and notice that we can write
L(P, f ) = L(Q1 , g1 ) + L(Q2 , g2 ) + mx0 (f ),
U (Q, f ) = U (Q1 , g1 ) + U (Q2 , g2 ) + Mx0 (f ),
where mx0 (f ) = inf{f (x) : x0 /2 x x0 + } and
Mx0 (f ) = sup{f (x) : x0 /2 x x0 + }. Since can be chosen as small as we
want then
Z x0
Z b
Z b
f dx =
g1 dx +
g2 dx
a

x0
x0

Z
f dx =

g1 dx +
a

g2 dx
x0

Therefore, f is Riemann integrable.

4. Suppose f is integrable on [a, b] and there exists k > 0 such that f (x) k for all
x [a, b]. Prove that 1/f is integrable.
Proof. Let > 0 and notice that
can find a partition P , such that

1
f (x)

1
k

for all x [a, b]. Since f is integrable, we

n1
X

U (P, f ) L(P, f ) =
(Mi (f ) mi (f ))(xi+1 xi ) < 2 .
k
i=0

Consider the function g : [a, b] R, g(x) =

1
. With the above partition we have
f (x)

that for each interval [xi , xi+1 ],


1
1
1
1

.
Mi (f )
f (x)
mi (f )
k

MATH 4603

Homework 9

Summer 2014

Moreover,
n1 
X


1
1
U (P, g) L(P, g) =

(xi+1 xi )
m
(f
)
M
(f
)
i
i
i=0

n1 
X
Mi (f ) mi (f )
=
(xi+1 xi )
M
(f
)m
(f
)
i
i
i=0

n1 
X
Mi (f ) mi (f )
<
(xi+1 xi )
2
k
i=0
< k 2 (U (P, f ) L(P, f )) < .
Therefore, g =

1
is Riemann integrable.
f

5. Suppose f is integrable on [b, b] and f is even. Show using Theorem 5.5 that
Rb
Rb
f dx = 2 0 f dx.
b
Proof. We will need to refer to the function g : [0, b] R, g(x) = f (x). We first
show that this function is integrable: Since f is integrable, given > 0 there is a
partition P of [b, b] such that U (P, f ) L(P, f ) < 2. Let
Q = {xi , xi : xi P } {0} be a refinement of P , and notice that
U (Q, f ) L(Q, f ) < 2. Furthermore, because f is even, if we denote by
= Q [0, b],
Q
g) 2L(Q,
g) < 2.
U (Q, f ) L(Q, f ) = 2U (Q,
g) L(Q,
g) < .
= U (Q,
Therefore, g is Riemann integrable.
Let > 0. Since g is Riemann integrable there is a partition P of [0, b] such that for
any refinement Q of P , regardless of how it is marked,
Z b
gdx| < /2.
|S(Q, g)
0

Also, because f is integrable there is a number A and a partition P 0 of [b, b] such


that for any refinement Q0 of P 0 , regardless of how it is marked,
|S(Q0 , f ) A| < /2.
Let P = P 0 P {0} and consider Q0 = {xi , xi : xi P } = {y0 , y1 , , yn }. Then
Q0 is a refinement of P 0 and Q = Q0 [0, b] is a refinement of P . Moreover,
0

S(Q , f ) =

n1
X
i=0

f (yi )(yi+1 yi ) = 2S(Q, g),

MATH 4603

Homework 9

so that
Z

Z
gdx| = |2S(Q, g) 2

|S(Q , f ) 2
Rb
b

f dx = 2

Rb
0

gdx = 2

gdx| <
0

Therefore,

Summer 2014

Rb
0

f dx.

6. Prove that f is integrable on [a, b] iff there exists a number A such that for every
> 0 there is a > 0 such that |S(Q, f ) A| < for every Riemann sum S(Q, f )
associated with a partition Q with mesh, (Q) < .
Proof. See Theorem 5.6

You might also like